Delta Epsilon Proof of $lim_{xtoinfty}frac{ln{x}^2}{x}=0$












6












$begingroup$


I am trying to prove that $lim_{xtoinfty}frac{ln{x}^2}{x}=0$ via the delta epsilon definition of a limit but I keep getting hung up.



I have started the proof like this:



$forall epsilon>0, exists delta$ so if $delta>0$, $|frac{ln{x}^2}{x}-0|<epsilon$.



If we suppose that $|frac{ln{x}^2}{x}-0|<epsilon$ and solve for x we get...



When I try solving the inequality for $x$, it keeps cancelling.



What I have tried:



$2x^{-1}ln{x}<ln{epsilon}$



Which becomes:



$2x^{-1}(x^1)<e^{ln{epsilon}}$



But then, $x^{-1}$ and $x^1$ multiply to $x^0$, which is $1$ and $x$ is removed from the problem.



Any pointers as how to proceed? I know that the limit is $0$, I just can't seem to get past solving for $x$.



Thank you.










share|cite|improve this question











$endgroup$












  • $begingroup$
    To typeset $x^{-1}$, use x^{-1}.
    $endgroup$
    – angryavian
    Dec 29 '18 at 2:32










  • $begingroup$
    Thank you @angryavian
    $endgroup$
    – limitsandlogs224
    Dec 29 '18 at 2:34










  • $begingroup$
    Hint: $ln (x^2)=2ln x$ for all $x>0.$
    $endgroup$
    – DanielWainfleet
    Dec 29 '18 at 6:36
















6












$begingroup$


I am trying to prove that $lim_{xtoinfty}frac{ln{x}^2}{x}=0$ via the delta epsilon definition of a limit but I keep getting hung up.



I have started the proof like this:



$forall epsilon>0, exists delta$ so if $delta>0$, $|frac{ln{x}^2}{x}-0|<epsilon$.



If we suppose that $|frac{ln{x}^2}{x}-0|<epsilon$ and solve for x we get...



When I try solving the inequality for $x$, it keeps cancelling.



What I have tried:



$2x^{-1}ln{x}<ln{epsilon}$



Which becomes:



$2x^{-1}(x^1)<e^{ln{epsilon}}$



But then, $x^{-1}$ and $x^1$ multiply to $x^0$, which is $1$ and $x$ is removed from the problem.



Any pointers as how to proceed? I know that the limit is $0$, I just can't seem to get past solving for $x$.



Thank you.










share|cite|improve this question











$endgroup$












  • $begingroup$
    To typeset $x^{-1}$, use x^{-1}.
    $endgroup$
    – angryavian
    Dec 29 '18 at 2:32










  • $begingroup$
    Thank you @angryavian
    $endgroup$
    – limitsandlogs224
    Dec 29 '18 at 2:34










  • $begingroup$
    Hint: $ln (x^2)=2ln x$ for all $x>0.$
    $endgroup$
    – DanielWainfleet
    Dec 29 '18 at 6:36














6












6








6





$begingroup$


I am trying to prove that $lim_{xtoinfty}frac{ln{x}^2}{x}=0$ via the delta epsilon definition of a limit but I keep getting hung up.



I have started the proof like this:



$forall epsilon>0, exists delta$ so if $delta>0$, $|frac{ln{x}^2}{x}-0|<epsilon$.



If we suppose that $|frac{ln{x}^2}{x}-0|<epsilon$ and solve for x we get...



When I try solving the inequality for $x$, it keeps cancelling.



What I have tried:



$2x^{-1}ln{x}<ln{epsilon}$



Which becomes:



$2x^{-1}(x^1)<e^{ln{epsilon}}$



But then, $x^{-1}$ and $x^1$ multiply to $x^0$, which is $1$ and $x$ is removed from the problem.



Any pointers as how to proceed? I know that the limit is $0$, I just can't seem to get past solving for $x$.



Thank you.










share|cite|improve this question











$endgroup$




I am trying to prove that $lim_{xtoinfty}frac{ln{x}^2}{x}=0$ via the delta epsilon definition of a limit but I keep getting hung up.



I have started the proof like this:



$forall epsilon>0, exists delta$ so if $delta>0$, $|frac{ln{x}^2}{x}-0|<epsilon$.



If we suppose that $|frac{ln{x}^2}{x}-0|<epsilon$ and solve for x we get...



When I try solving the inequality for $x$, it keeps cancelling.



What I have tried:



$2x^{-1}ln{x}<ln{epsilon}$



Which becomes:



$2x^{-1}(x^1)<e^{ln{epsilon}}$



But then, $x^{-1}$ and $x^1$ multiply to $x^0$, which is $1$ and $x$ is removed from the problem.



Any pointers as how to proceed? I know that the limit is $0$, I just can't seem to get past solving for $x$.



Thank you.







calculus proof-verification epsilon-delta






share|cite|improve this question















share|cite|improve this question













share|cite|improve this question




share|cite|improve this question








edited Dec 29 '18 at 5:09









The Count

2,31361431




2,31361431










asked Dec 29 '18 at 2:26









limitsandlogs224limitsandlogs224

576




576












  • $begingroup$
    To typeset $x^{-1}$, use x^{-1}.
    $endgroup$
    – angryavian
    Dec 29 '18 at 2:32










  • $begingroup$
    Thank you @angryavian
    $endgroup$
    – limitsandlogs224
    Dec 29 '18 at 2:34










  • $begingroup$
    Hint: $ln (x^2)=2ln x$ for all $x>0.$
    $endgroup$
    – DanielWainfleet
    Dec 29 '18 at 6:36


















  • $begingroup$
    To typeset $x^{-1}$, use x^{-1}.
    $endgroup$
    – angryavian
    Dec 29 '18 at 2:32










  • $begingroup$
    Thank you @angryavian
    $endgroup$
    – limitsandlogs224
    Dec 29 '18 at 2:34










  • $begingroup$
    Hint: $ln (x^2)=2ln x$ for all $x>0.$
    $endgroup$
    – DanielWainfleet
    Dec 29 '18 at 6:36
















$begingroup$
To typeset $x^{-1}$, use x^{-1}.
$endgroup$
– angryavian
Dec 29 '18 at 2:32




$begingroup$
To typeset $x^{-1}$, use x^{-1}.
$endgroup$
– angryavian
Dec 29 '18 at 2:32












$begingroup$
Thank you @angryavian
$endgroup$
– limitsandlogs224
Dec 29 '18 at 2:34




$begingroup$
Thank you @angryavian
$endgroup$
– limitsandlogs224
Dec 29 '18 at 2:34












$begingroup$
Hint: $ln (x^2)=2ln x$ for all $x>0.$
$endgroup$
– DanielWainfleet
Dec 29 '18 at 6:36




$begingroup$
Hint: $ln (x^2)=2ln x$ for all $x>0.$
$endgroup$
– DanielWainfleet
Dec 29 '18 at 6:36










2 Answers
2






active

oldest

votes


















2












$begingroup$


... I have started the proof like this:



$forall epsilon>0, exists delta$ so if $delta>0$, $|frac{ln{x}^2}{x}-0|<epsilon$.




The $delta$ in your attempt has nothing to do with $epsilon$.



By definition, given $epsilon>0$, you need to show that there exists $M>0$ such that
$$
x>M quad textbf{implies }quad |frac{2ln x}{x}|<epsilontag{1}
$$



Now for $x>1$,
$$
ln x=int_1^xfrac{1}{t} dtleq int_1^xfrac{1}{sqrt{t}} dt=2sqrt{t}big|_1^x
< 2sqrt{x},
$$

which implies that
$$
frac{2ln x}{x}<frac{4sqrt{x}}{x}=frac{4}{sqrt{x}},quad x>1.
$$

So you want
$
frac{4}{sqrt{x}}<epsilon,
$

which is true if
$$
x>(4/epsilon)^2.
$$

Now, setting
$M=(4/epsilon)^2$, you have the implication (1).






share|cite|improve this answer









$endgroup$









  • 3




    $begingroup$
    Clever way of showing $ln x<2sqrt x$ for $x>1$.
    $endgroup$
    – DanielWainfleet
    Dec 29 '18 at 6:39










  • $begingroup$
    What does $|_1^x$ mean?
    $endgroup$
    – limitsandlogs224
    Dec 31 '18 at 1:12










  • $begingroup$
    @limitsandlogs224: $f(x)|_a^b:=f(b)-f(a)$.
    $endgroup$
    – user587192
    Dec 31 '18 at 1:53



















4












$begingroup$

$ln x<2sqrt{x}$ so
$$left|dfrac{ln x^2}{x}right|<dfrac{4}{sqrt{x}}<dfrac{4}{sqrt{M}}leqvarepsilon$$
where $x>M$.






share|cite|improve this answer









$endgroup$













  • $begingroup$
    How do you prove $ln x< 2sqrt{x}$?
    $endgroup$
    – user587192
    Dec 29 '18 at 2:52










  • $begingroup$
    $ln x<dfrac{x^alpha}{alpha}$ @user587192 with $alpha>0$.
    $endgroup$
    – Nosrati
    Dec 29 '18 at 3:01












  • $begingroup$
    Proof: $e^u>u$ therefore $u>ln u$ let $u=x^alpha$.
    $endgroup$
    – Nosrati
    Dec 29 '18 at 3:15










  • $begingroup$
    @Nosrati, thank you. Could you please explain how I could incorporate $frac{4}{x^{frac{1}{2}}}$ and $frac{4}{M^{frac{1}{2}}}$ into the proof? I thought that I had to solve $frac{4}{x{frac{1}{2}}} <epsilon$ for $x$ so any $epsilon$ is $>|frac{ln{x}^2}{x}-0|$ as long as $delta>frac{16}{epsilon^2}$. I see that they keep $x$ from getting eliminated but I just don't understand how those expressions get substituted into the inequality.
    $endgroup$
    – limitsandlogs224
    Dec 29 '18 at 3:45








  • 1




    $begingroup$
    In fact, it is not difficult to show that $log(x)<sqrt x$ for all $x>0$. Going one step further $log(x)le x^{1/e}$ for all $x$ with equality at $e^e$.
    $endgroup$
    – Mark Viola
    Dec 29 '18 at 20:20













Your Answer





StackExchange.ifUsing("editor", function () {
return StackExchange.using("mathjaxEditing", function () {
StackExchange.MarkdownEditor.creationCallbacks.add(function (editor, postfix) {
StackExchange.mathjaxEditing.prepareWmdForMathJax(editor, postfix, [["$", "$"], ["\\(","\\)"]]);
});
});
}, "mathjax-editing");

StackExchange.ready(function() {
var channelOptions = {
tags: "".split(" "),
id: "69"
};
initTagRenderer("".split(" "), "".split(" "), channelOptions);

StackExchange.using("externalEditor", function() {
// Have to fire editor after snippets, if snippets enabled
if (StackExchange.settings.snippets.snippetsEnabled) {
StackExchange.using("snippets", function() {
createEditor();
});
}
else {
createEditor();
}
});

function createEditor() {
StackExchange.prepareEditor({
heartbeatType: 'answer',
autoActivateHeartbeat: false,
convertImagesToLinks: true,
noModals: true,
showLowRepImageUploadWarning: true,
reputationToPostImages: 10,
bindNavPrevention: true,
postfix: "",
imageUploader: {
brandingHtml: "Powered by u003ca class="icon-imgur-white" href="https://imgur.com/"u003eu003c/au003e",
contentPolicyHtml: "User contributions licensed under u003ca href="https://creativecommons.org/licenses/by-sa/3.0/"u003ecc by-sa 3.0 with attribution requiredu003c/au003e u003ca href="https://stackoverflow.com/legal/content-policy"u003e(content policy)u003c/au003e",
allowUrls: true
},
noCode: true, onDemand: true,
discardSelector: ".discard-answer"
,immediatelyShowMarkdownHelp:true
});


}
});














draft saved

draft discarded


















StackExchange.ready(
function () {
StackExchange.openid.initPostLogin('.new-post-login', 'https%3a%2f%2fmath.stackexchange.com%2fquestions%2f3055480%2fdelta-epsilon-proof-of-lim-x-to-infty-frac-lnx2x-0%23new-answer', 'question_page');
}
);

Post as a guest















Required, but never shown

























2 Answers
2






active

oldest

votes








2 Answers
2






active

oldest

votes









active

oldest

votes






active

oldest

votes









2












$begingroup$


... I have started the proof like this:



$forall epsilon>0, exists delta$ so if $delta>0$, $|frac{ln{x}^2}{x}-0|<epsilon$.




The $delta$ in your attempt has nothing to do with $epsilon$.



By definition, given $epsilon>0$, you need to show that there exists $M>0$ such that
$$
x>M quad textbf{implies }quad |frac{2ln x}{x}|<epsilontag{1}
$$



Now for $x>1$,
$$
ln x=int_1^xfrac{1}{t} dtleq int_1^xfrac{1}{sqrt{t}} dt=2sqrt{t}big|_1^x
< 2sqrt{x},
$$

which implies that
$$
frac{2ln x}{x}<frac{4sqrt{x}}{x}=frac{4}{sqrt{x}},quad x>1.
$$

So you want
$
frac{4}{sqrt{x}}<epsilon,
$

which is true if
$$
x>(4/epsilon)^2.
$$

Now, setting
$M=(4/epsilon)^2$, you have the implication (1).






share|cite|improve this answer









$endgroup$









  • 3




    $begingroup$
    Clever way of showing $ln x<2sqrt x$ for $x>1$.
    $endgroup$
    – DanielWainfleet
    Dec 29 '18 at 6:39










  • $begingroup$
    What does $|_1^x$ mean?
    $endgroup$
    – limitsandlogs224
    Dec 31 '18 at 1:12










  • $begingroup$
    @limitsandlogs224: $f(x)|_a^b:=f(b)-f(a)$.
    $endgroup$
    – user587192
    Dec 31 '18 at 1:53
















2












$begingroup$


... I have started the proof like this:



$forall epsilon>0, exists delta$ so if $delta>0$, $|frac{ln{x}^2}{x}-0|<epsilon$.




The $delta$ in your attempt has nothing to do with $epsilon$.



By definition, given $epsilon>0$, you need to show that there exists $M>0$ such that
$$
x>M quad textbf{implies }quad |frac{2ln x}{x}|<epsilontag{1}
$$



Now for $x>1$,
$$
ln x=int_1^xfrac{1}{t} dtleq int_1^xfrac{1}{sqrt{t}} dt=2sqrt{t}big|_1^x
< 2sqrt{x},
$$

which implies that
$$
frac{2ln x}{x}<frac{4sqrt{x}}{x}=frac{4}{sqrt{x}},quad x>1.
$$

So you want
$
frac{4}{sqrt{x}}<epsilon,
$

which is true if
$$
x>(4/epsilon)^2.
$$

Now, setting
$M=(4/epsilon)^2$, you have the implication (1).






share|cite|improve this answer









$endgroup$









  • 3




    $begingroup$
    Clever way of showing $ln x<2sqrt x$ for $x>1$.
    $endgroup$
    – DanielWainfleet
    Dec 29 '18 at 6:39










  • $begingroup$
    What does $|_1^x$ mean?
    $endgroup$
    – limitsandlogs224
    Dec 31 '18 at 1:12










  • $begingroup$
    @limitsandlogs224: $f(x)|_a^b:=f(b)-f(a)$.
    $endgroup$
    – user587192
    Dec 31 '18 at 1:53














2












2








2





$begingroup$


... I have started the proof like this:



$forall epsilon>0, exists delta$ so if $delta>0$, $|frac{ln{x}^2}{x}-0|<epsilon$.




The $delta$ in your attempt has nothing to do with $epsilon$.



By definition, given $epsilon>0$, you need to show that there exists $M>0$ such that
$$
x>M quad textbf{implies }quad |frac{2ln x}{x}|<epsilontag{1}
$$



Now for $x>1$,
$$
ln x=int_1^xfrac{1}{t} dtleq int_1^xfrac{1}{sqrt{t}} dt=2sqrt{t}big|_1^x
< 2sqrt{x},
$$

which implies that
$$
frac{2ln x}{x}<frac{4sqrt{x}}{x}=frac{4}{sqrt{x}},quad x>1.
$$

So you want
$
frac{4}{sqrt{x}}<epsilon,
$

which is true if
$$
x>(4/epsilon)^2.
$$

Now, setting
$M=(4/epsilon)^2$, you have the implication (1).






share|cite|improve this answer









$endgroup$




... I have started the proof like this:



$forall epsilon>0, exists delta$ so if $delta>0$, $|frac{ln{x}^2}{x}-0|<epsilon$.




The $delta$ in your attempt has nothing to do with $epsilon$.



By definition, given $epsilon>0$, you need to show that there exists $M>0$ such that
$$
x>M quad textbf{implies }quad |frac{2ln x}{x}|<epsilontag{1}
$$



Now for $x>1$,
$$
ln x=int_1^xfrac{1}{t} dtleq int_1^xfrac{1}{sqrt{t}} dt=2sqrt{t}big|_1^x
< 2sqrt{x},
$$

which implies that
$$
frac{2ln x}{x}<frac{4sqrt{x}}{x}=frac{4}{sqrt{x}},quad x>1.
$$

So you want
$
frac{4}{sqrt{x}}<epsilon,
$

which is true if
$$
x>(4/epsilon)^2.
$$

Now, setting
$M=(4/epsilon)^2$, you have the implication (1).







share|cite|improve this answer












share|cite|improve this answer



share|cite|improve this answer










answered Dec 29 '18 at 4:55







user587192















  • 3




    $begingroup$
    Clever way of showing $ln x<2sqrt x$ for $x>1$.
    $endgroup$
    – DanielWainfleet
    Dec 29 '18 at 6:39










  • $begingroup$
    What does $|_1^x$ mean?
    $endgroup$
    – limitsandlogs224
    Dec 31 '18 at 1:12










  • $begingroup$
    @limitsandlogs224: $f(x)|_a^b:=f(b)-f(a)$.
    $endgroup$
    – user587192
    Dec 31 '18 at 1:53














  • 3




    $begingroup$
    Clever way of showing $ln x<2sqrt x$ for $x>1$.
    $endgroup$
    – DanielWainfleet
    Dec 29 '18 at 6:39










  • $begingroup$
    What does $|_1^x$ mean?
    $endgroup$
    – limitsandlogs224
    Dec 31 '18 at 1:12










  • $begingroup$
    @limitsandlogs224: $f(x)|_a^b:=f(b)-f(a)$.
    $endgroup$
    – user587192
    Dec 31 '18 at 1:53








3




3




$begingroup$
Clever way of showing $ln x<2sqrt x$ for $x>1$.
$endgroup$
– DanielWainfleet
Dec 29 '18 at 6:39




$begingroup$
Clever way of showing $ln x<2sqrt x$ for $x>1$.
$endgroup$
– DanielWainfleet
Dec 29 '18 at 6:39












$begingroup$
What does $|_1^x$ mean?
$endgroup$
– limitsandlogs224
Dec 31 '18 at 1:12




$begingroup$
What does $|_1^x$ mean?
$endgroup$
– limitsandlogs224
Dec 31 '18 at 1:12












$begingroup$
@limitsandlogs224: $f(x)|_a^b:=f(b)-f(a)$.
$endgroup$
– user587192
Dec 31 '18 at 1:53




$begingroup$
@limitsandlogs224: $f(x)|_a^b:=f(b)-f(a)$.
$endgroup$
– user587192
Dec 31 '18 at 1:53











4












$begingroup$

$ln x<2sqrt{x}$ so
$$left|dfrac{ln x^2}{x}right|<dfrac{4}{sqrt{x}}<dfrac{4}{sqrt{M}}leqvarepsilon$$
where $x>M$.






share|cite|improve this answer









$endgroup$













  • $begingroup$
    How do you prove $ln x< 2sqrt{x}$?
    $endgroup$
    – user587192
    Dec 29 '18 at 2:52










  • $begingroup$
    $ln x<dfrac{x^alpha}{alpha}$ @user587192 with $alpha>0$.
    $endgroup$
    – Nosrati
    Dec 29 '18 at 3:01












  • $begingroup$
    Proof: $e^u>u$ therefore $u>ln u$ let $u=x^alpha$.
    $endgroup$
    – Nosrati
    Dec 29 '18 at 3:15










  • $begingroup$
    @Nosrati, thank you. Could you please explain how I could incorporate $frac{4}{x^{frac{1}{2}}}$ and $frac{4}{M^{frac{1}{2}}}$ into the proof? I thought that I had to solve $frac{4}{x{frac{1}{2}}} <epsilon$ for $x$ so any $epsilon$ is $>|frac{ln{x}^2}{x}-0|$ as long as $delta>frac{16}{epsilon^2}$. I see that they keep $x$ from getting eliminated but I just don't understand how those expressions get substituted into the inequality.
    $endgroup$
    – limitsandlogs224
    Dec 29 '18 at 3:45








  • 1




    $begingroup$
    In fact, it is not difficult to show that $log(x)<sqrt x$ for all $x>0$. Going one step further $log(x)le x^{1/e}$ for all $x$ with equality at $e^e$.
    $endgroup$
    – Mark Viola
    Dec 29 '18 at 20:20


















4












$begingroup$

$ln x<2sqrt{x}$ so
$$left|dfrac{ln x^2}{x}right|<dfrac{4}{sqrt{x}}<dfrac{4}{sqrt{M}}leqvarepsilon$$
where $x>M$.






share|cite|improve this answer









$endgroup$













  • $begingroup$
    How do you prove $ln x< 2sqrt{x}$?
    $endgroup$
    – user587192
    Dec 29 '18 at 2:52










  • $begingroup$
    $ln x<dfrac{x^alpha}{alpha}$ @user587192 with $alpha>0$.
    $endgroup$
    – Nosrati
    Dec 29 '18 at 3:01












  • $begingroup$
    Proof: $e^u>u$ therefore $u>ln u$ let $u=x^alpha$.
    $endgroup$
    – Nosrati
    Dec 29 '18 at 3:15










  • $begingroup$
    @Nosrati, thank you. Could you please explain how I could incorporate $frac{4}{x^{frac{1}{2}}}$ and $frac{4}{M^{frac{1}{2}}}$ into the proof? I thought that I had to solve $frac{4}{x{frac{1}{2}}} <epsilon$ for $x$ so any $epsilon$ is $>|frac{ln{x}^2}{x}-0|$ as long as $delta>frac{16}{epsilon^2}$. I see that they keep $x$ from getting eliminated but I just don't understand how those expressions get substituted into the inequality.
    $endgroup$
    – limitsandlogs224
    Dec 29 '18 at 3:45








  • 1




    $begingroup$
    In fact, it is not difficult to show that $log(x)<sqrt x$ for all $x>0$. Going one step further $log(x)le x^{1/e}$ for all $x$ with equality at $e^e$.
    $endgroup$
    – Mark Viola
    Dec 29 '18 at 20:20
















4












4








4





$begingroup$

$ln x<2sqrt{x}$ so
$$left|dfrac{ln x^2}{x}right|<dfrac{4}{sqrt{x}}<dfrac{4}{sqrt{M}}leqvarepsilon$$
where $x>M$.






share|cite|improve this answer









$endgroup$



$ln x<2sqrt{x}$ so
$$left|dfrac{ln x^2}{x}right|<dfrac{4}{sqrt{x}}<dfrac{4}{sqrt{M}}leqvarepsilon$$
where $x>M$.







share|cite|improve this answer












share|cite|improve this answer



share|cite|improve this answer










answered Dec 29 '18 at 2:40









NosratiNosrati

26.5k62354




26.5k62354












  • $begingroup$
    How do you prove $ln x< 2sqrt{x}$?
    $endgroup$
    – user587192
    Dec 29 '18 at 2:52










  • $begingroup$
    $ln x<dfrac{x^alpha}{alpha}$ @user587192 with $alpha>0$.
    $endgroup$
    – Nosrati
    Dec 29 '18 at 3:01












  • $begingroup$
    Proof: $e^u>u$ therefore $u>ln u$ let $u=x^alpha$.
    $endgroup$
    – Nosrati
    Dec 29 '18 at 3:15










  • $begingroup$
    @Nosrati, thank you. Could you please explain how I could incorporate $frac{4}{x^{frac{1}{2}}}$ and $frac{4}{M^{frac{1}{2}}}$ into the proof? I thought that I had to solve $frac{4}{x{frac{1}{2}}} <epsilon$ for $x$ so any $epsilon$ is $>|frac{ln{x}^2}{x}-0|$ as long as $delta>frac{16}{epsilon^2}$. I see that they keep $x$ from getting eliminated but I just don't understand how those expressions get substituted into the inequality.
    $endgroup$
    – limitsandlogs224
    Dec 29 '18 at 3:45








  • 1




    $begingroup$
    In fact, it is not difficult to show that $log(x)<sqrt x$ for all $x>0$. Going one step further $log(x)le x^{1/e}$ for all $x$ with equality at $e^e$.
    $endgroup$
    – Mark Viola
    Dec 29 '18 at 20:20




















  • $begingroup$
    How do you prove $ln x< 2sqrt{x}$?
    $endgroup$
    – user587192
    Dec 29 '18 at 2:52










  • $begingroup$
    $ln x<dfrac{x^alpha}{alpha}$ @user587192 with $alpha>0$.
    $endgroup$
    – Nosrati
    Dec 29 '18 at 3:01












  • $begingroup$
    Proof: $e^u>u$ therefore $u>ln u$ let $u=x^alpha$.
    $endgroup$
    – Nosrati
    Dec 29 '18 at 3:15










  • $begingroup$
    @Nosrati, thank you. Could you please explain how I could incorporate $frac{4}{x^{frac{1}{2}}}$ and $frac{4}{M^{frac{1}{2}}}$ into the proof? I thought that I had to solve $frac{4}{x{frac{1}{2}}} <epsilon$ for $x$ so any $epsilon$ is $>|frac{ln{x}^2}{x}-0|$ as long as $delta>frac{16}{epsilon^2}$. I see that they keep $x$ from getting eliminated but I just don't understand how those expressions get substituted into the inequality.
    $endgroup$
    – limitsandlogs224
    Dec 29 '18 at 3:45








  • 1




    $begingroup$
    In fact, it is not difficult to show that $log(x)<sqrt x$ for all $x>0$. Going one step further $log(x)le x^{1/e}$ for all $x$ with equality at $e^e$.
    $endgroup$
    – Mark Viola
    Dec 29 '18 at 20:20


















$begingroup$
How do you prove $ln x< 2sqrt{x}$?
$endgroup$
– user587192
Dec 29 '18 at 2:52




$begingroup$
How do you prove $ln x< 2sqrt{x}$?
$endgroup$
– user587192
Dec 29 '18 at 2:52












$begingroup$
$ln x<dfrac{x^alpha}{alpha}$ @user587192 with $alpha>0$.
$endgroup$
– Nosrati
Dec 29 '18 at 3:01






$begingroup$
$ln x<dfrac{x^alpha}{alpha}$ @user587192 with $alpha>0$.
$endgroup$
– Nosrati
Dec 29 '18 at 3:01














$begingroup$
Proof: $e^u>u$ therefore $u>ln u$ let $u=x^alpha$.
$endgroup$
– Nosrati
Dec 29 '18 at 3:15




$begingroup$
Proof: $e^u>u$ therefore $u>ln u$ let $u=x^alpha$.
$endgroup$
– Nosrati
Dec 29 '18 at 3:15












$begingroup$
@Nosrati, thank you. Could you please explain how I could incorporate $frac{4}{x^{frac{1}{2}}}$ and $frac{4}{M^{frac{1}{2}}}$ into the proof? I thought that I had to solve $frac{4}{x{frac{1}{2}}} <epsilon$ for $x$ so any $epsilon$ is $>|frac{ln{x}^2}{x}-0|$ as long as $delta>frac{16}{epsilon^2}$. I see that they keep $x$ from getting eliminated but I just don't understand how those expressions get substituted into the inequality.
$endgroup$
– limitsandlogs224
Dec 29 '18 at 3:45






$begingroup$
@Nosrati, thank you. Could you please explain how I could incorporate $frac{4}{x^{frac{1}{2}}}$ and $frac{4}{M^{frac{1}{2}}}$ into the proof? I thought that I had to solve $frac{4}{x{frac{1}{2}}} <epsilon$ for $x$ so any $epsilon$ is $>|frac{ln{x}^2}{x}-0|$ as long as $delta>frac{16}{epsilon^2}$. I see that they keep $x$ from getting eliminated but I just don't understand how those expressions get substituted into the inequality.
$endgroup$
– limitsandlogs224
Dec 29 '18 at 3:45






1




1




$begingroup$
In fact, it is not difficult to show that $log(x)<sqrt x$ for all $x>0$. Going one step further $log(x)le x^{1/e}$ for all $x$ with equality at $e^e$.
$endgroup$
– Mark Viola
Dec 29 '18 at 20:20






$begingroup$
In fact, it is not difficult to show that $log(x)<sqrt x$ for all $x>0$. Going one step further $log(x)le x^{1/e}$ for all $x$ with equality at $e^e$.
$endgroup$
– Mark Viola
Dec 29 '18 at 20:20




















draft saved

draft discarded




















































Thanks for contributing an answer to Mathematics Stack Exchange!


  • Please be sure to answer the question. Provide details and share your research!

But avoid



  • Asking for help, clarification, or responding to other answers.

  • Making statements based on opinion; back them up with references or personal experience.


Use MathJax to format equations. MathJax reference.


To learn more, see our tips on writing great answers.




draft saved


draft discarded














StackExchange.ready(
function () {
StackExchange.openid.initPostLogin('.new-post-login', 'https%3a%2f%2fmath.stackexchange.com%2fquestions%2f3055480%2fdelta-epsilon-proof-of-lim-x-to-infty-frac-lnx2x-0%23new-answer', 'question_page');
}
);

Post as a guest















Required, but never shown





















































Required, but never shown














Required, but never shown












Required, but never shown







Required, but never shown

































Required, but never shown














Required, but never shown












Required, but never shown







Required, but never shown







Popular posts from this blog

How do I know what Microsoft account the skydrive app is syncing to?

When does type information flow backwards in C++?

Grease: Live!